Difference between revisions of "2006 USAMO Problems/Problem 4"

(Solution)
(My solution was wrong wrong wrong, after reading the topic.)
Line 4: Line 4:
  
 
== Solution ==
 
== Solution ==
Let polynomial <math>P(x)</math> be a [[monic polynomial]] such that it's roots are only all of <math>a_1</math> through <math>a_k</math>. Therefore, the sum and product of the roots is n, and the constant term of <math>P(x)</math> is <math>\pm n</math>. From the [[Rational Root Theorem]], all <math>a_i</math> are divisors of n, and integral. We split this into cases:
+
{{solution}}
 
 
 
 
 
 
''Case 1: n is prime''
 
 
 
If n is prime, the only divisors of n are 1 and n. We must have an n in <math>a_i</math> so that <math>a_1 \cdot a_2 \cdot \cdots a_k = n</math>, but then <math>a_1+a_2+...+a_k>n</math>, since <math>k\geq 1</math>. We have a contradiction, therefore n may not be prime.
 
 
 
 
 
 
 
''Case 2: n is composite''
 
 
 
Let two divisors of n(not necessarily distinct) be <math>d_1</math> and <math>d_2</math>, such that <math>d_1\cdot d_2 = n</math> and <math>d_1,d_2<n</math>. We will prove that <math>d_1+d_2 \leq n</math>:
 
 
 
We subtract <math>d_1+d_2</math> from <math>d_1d_2</math>: <math>d_1d_2-d_1-d_2</math>. Now we add 1: <math>d_1d_2-d_1-d_2+1=(d_1-1)(d_2-1)</math>. Since <math>d_1</math> and <math>d_2</math> are greater than 1, <math>d_1 -1</math> and <math>d_2 -1</math> are positive. Therefore, <math>d_1d_2\geq d_1+d_2</math>, and <math>d_1+d_2\leq n</math>.
 
 
 
[[WLOG]], we let <math>a_1=d_1</math> and <math>a_2=d_2</math>. If <math>n-a_1-a_2>0</math>, we can let the rest of the numbers be ones. Therefore, there are such k when n is composite.
 
 
 
 
 
 
 
''Case 3: n=1''
 
 
 
Therefore, k=1, but <math>k\geq 2</math>, so that is impossible.
 
 
 
 
 
 
 
Therefore, there are such <math>k\geq 2</math> such that <math>a_1+a_2+...+a_k = a_1 \cdot a_2 \cdot \cdots a_k = n</math> only when n is composite.
 
  
 
== See Also ==
 
== See Also ==

Revision as of 12:12, 28 January 2008

Problem

Find all positive integers $n$ such that there are $k\ge 2$ positive rational numbers $a_1,a_2,\ldots a_k$ satisfying $a_1+a_2+...+a_k = a_1 \cdot a_2 \cdot \cdots a_k = n$.

Solution

This problem needs a solution. If you have a solution for it, please help us out by adding it.

See Also